4
$\begingroup$

EDIT The initial revision had typo, hope it is fixed.

Let $C$ be the hyperelliptic curve $y^2=x^5+x+5^2$ and $J$ its Jacobian.

Let $P=J(0,5)$ and define $a(n)$ to be the constant coefficient of the first coordinate of $nP$.

$a(n)$ starts $1, 0, 0, 1/5000, 12500, 200000003/125000$.

Experimentally $a(n)$ satisfy the nonlinear recurrence up to $n=100$:

a(n + 2) = -((50000002*a(n - 1)*a(n - 2) - 10000*a(n - 2) - 5)*a(n + 1)*a(n) - 5*(2000*a(n - 2) + 1)*a(n - 1)*a(n) - 5*a(n - 1)*a(n - 2) + 25000*a(n - 2) + 100000014)/(10000*(5000*a(n - 2) - 1)*a(n - 1)*a(n) + (50000002*a(n - 1)*a(n - 2) + 2*(25000001*a(n - 1) + 25000000*a(n - 2) - 5000)*a(n) - 10000*a(n - 2) - 5)*a(n + 1) - 10000*a(n - 1)*a(n - 2) + 25000)

Is the recurrence true?

sagemath code:

def jacob1(KK,lim=100):
    """
    K=QQ
    """
    pr=PolynomialRing(KK,'x')
    x=pr.gen()
    w0=KK(5)
    dg=5
    pol= x**5+x+5**2  
    C = HyperellipticCurve(pol)
    pt0=C([KK(0),KK(5)**1,KK(1)])
    J=C.jacobian()
    P=J(KK)(pt0)
    f=[]
    for i in srange(lim):
        Q= i*P
        u=Q[0]
        T=u.constant_coefficient()
        f.append(T)
        print u
        print '  ',T
    return f
$\endgroup$
10
  • 2
    $\begingroup$ Maybe this is standard terminology, but I don't understand...$nP$ is a point on $J$, so what does "first coordinate of $nP$" mean? You also haven't specified how you've chosen to embed $C$ in $J$... $\endgroup$ Oct 24, 2017 at 14:33
  • $\begingroup$ @AriShnidman $nP=(u,v)$ and the first coordinate is $u$. Sagemath returns J as C.jacobian(). $\endgroup$
    – joro
    Oct 24, 2017 at 14:39
  • $\begingroup$ I assume that this is the Mumford representation: $u$ and $v$ are polynomials; $nP$ is the sum of the points whose $x$-coordinates are roots of $u$ and whose $y$-coordinates are obtained by evaluating $v$ at the $x$-coordinate (and we use the point at infinity as the base-point for embedding $C$ into $J$). However, I get different values for the constant terms of $u_n$ (where $nP = (u_n,v_n)$: $1, 0, 0, 1/128, 320, 5243/128, -219962722816/687226225, \ldots$ (using Magma). $\endgroup$ Oct 24, 2017 at 18:08
  • $\begingroup$ Please tell us how you computed $nP=(u_n,v_n)$ and give the first few values of both $u_n$ and $v_n$. $\endgroup$
    – Somos
    Oct 24, 2017 at 19:41
  • $\begingroup$ @MichaelStoll I had typo, was working with the curve y^2=x^5+x+5^2. Fixed it and added sagemath code. $\endgroup$
    – joro
    Oct 25, 2017 at 6:40

1 Answer 1

9
$\begingroup$

NEW (Oct 27, 2017)

I can now show that the recurrence given in the Question is correct. In fact, I can deduce a simpler and shorter recurrence relation.

For this, we consider the Kummer Surface $K$ associated to $J$. This is a quartic surface in ${\mathbb P}^3$, and there is a map $\kappa \colon J \to K$ that identifies a point $P$ and its negative $-P$ (so it is 2-to-1 except at the 16 points of order 2 on $J$). In terms of the coordinates one usually uses, the image of your point on $K$ is $(0:1:0:0)$. Also, the constant term in the first component of the Mumford representation of a point with image $(x_1:x_2:x_3:x_4)$ on $K$ is $x_3/x_1$ (assuming $x_1 \neq 0$). Now there is a $4 \times 4$ matrix $B(\vec{x},\vec{y})$ of bi-quadratic forms in two sets of variables $\vec{x} = (x_1,\ldots,x_4)$ and $\vec{y} = (y_1,\ldots,y_4)$ (i.e., separately homogeneous of degree 2 in each set of variables) such that, if $\vec{x},\vec{y},\vec{w},\vec{z}$ are coordinate vectors of the images of points $P, Q, P-Q, P+Q$, resp., then, up to a common scaling factor, $$B_{ii}(\vec{x},\vec{y}) = w_i z_i \quad\text{and}\quad B_{ij}(\vec{x},\vec{y}) = w_i z_j + w_j z_i \quad\text{for $i \neq j$.}$$ (See the book by Cassels and Flynn for background.) We write $\kappa(nP) = (1 : x_{n,2} : x_{n,3} : x_{n,4})$ (assuming the first coordinate is non-zero), so that $x_{n,3}$ is the term of interest. It is easy to see that the matrix $B(\vec{x},\vec{y})$ determines the unordered pair $\{\vec{w}, \vec{z}\}$ (up to scaling of the coordinate vectors).

Now we use the relations $B(\vec{x}_n, \vec{x}_1) \sim \vec{x}_{n-1} * \vec{x}_{n+1}$ (where $\vec{w} * \vec{z}$ denotes the symmetric matrix with entries $w_i z_i$ on the diagonal and $w_i z_j + w_j z_i$ off the diagonal) for two successive values of $n$, together with the equations $f(\vec{x}_{n-1}) = \ldots = f(\vec{x}_{n+2}) = 0$, where $f = 0$ is the equation defining $K$. Together they define an ideal in the polynomial ring in 12 variables over $\mathbb Q$, and we can compute the elimination ideal $I$ with respect to $\{x_{n-1,3}, x_{n,3}, x_{n+1,3}, x_{n+2,3}\}$. Magma says that $I$ is generated by 10 polynomials, one of which is linear in $x_{n+2,3}$ (and also in $x_{n-1,3}$). Writing $a_n$ for $x_{n,3}$ and solving, we find the three-term recurrence $$a_{n+2} = a_n \frac{a_{n-1}+a_{n+1}}{a_{n-1} - a_{n+1}} - 2 \frac{a_n^2 a_{n+1}^2 + a_n a_{n+1} + 5000(a_n + a_{n+1}) - 1} {(a_{n-1}-a_{n+1}) a_n^2 a_{n+1}^2} $$

We can also check that the four-term recurrence given in the question (in terms of the algebraic relation it implies between five successive values of $a_{n}$) is implied by the relations that hold between five successive coordinate vectors.

In the spirit of a previous remark (that I removed when re-writing the answer to give a complete solution), note that the variety $V_k$ given by $$B(\vec{x}_n,\vec{x}_1) \sim \vec{x}_{n-1} * \vec{x}_{n+1}, \ldots, B(\vec{x}_{n+k},\vec{x}_1) \sim \vec{x}_{n+k-1} * \vec{x}_{n+k+1}$$ and $$f(\vec{x}_{n-1}) = \ldots = f(\vec{x}_{n+k+1}) = 0$$ is two-dimensional for any $k \ge 0$. So we can expect a relation between any three quantities depending on the variables. For example, there is a relation relating $a_{n-1}$, $a_n$ and $a_{n+1}$, but it is of degree 4 in $a_{n-1}$ and in $a_{n+1}$ and of degree 6 in $a_n$. We need to consider more quantities to find a relation that is linear in one of them.

$\endgroup$
2
  • $\begingroup$ Is it expected $u_n(1)$ to satisfy something similar? $\endgroup$
    – joro
    Oct 26, 2017 at 9:23
  • $\begingroup$ Did induction fail to show equivalence of the recurrences? $\endgroup$
    – joro
    Oct 27, 2017 at 14:25

Your Answer

By clicking “Post Your Answer”, you agree to our terms of service and acknowledge you have read our privacy policy.

Not the answer you're looking for? Browse other questions tagged or ask your own question.